Đến nội dung

taideptrai nội dung

Có 99 mục bởi taideptrai (Tìm giới hạn từ 06-06-2020)



Sắp theo                Sắp xếp  

#532492 Đề thi chọn đội tuyển HSG QG Hà Nội năm học 2014-2015

Đã gửi bởi taideptrai on 09-11-2014 - 10:46 trong Thi HSG cấp Tỉnh, Thành phố. Olympic 30-4. Đề thi và kiểm tra đội tuyển các cấp.

Dùng fermat để CM bổ đề sau với x,y không cùng chia hết cho ước số nào có dạng 4k-1 thì $x^{2}+y^{2}$ cũng ko có ước dạng 4k-1.

cái đấy là để cm chiều thuận, còn chiều đảo mới khó, cậu thử cm chiều đảo đi




#532398 Đề thi chọn đội tuyển HSG QG Hà Nội năm học 2014-2015

Đã gửi bởi taideptrai on 08-11-2014 - 20:25 trong Thi HSG cấp Tỉnh, Thành phố. Olympic 30-4. Đề thi và kiểm tra đội tuyển các cấp.

Ta sẽ tìm m bằng phương pháp quy nạp:

gọi m là số sao cho $m^{2}\equiv -9 (mod 2^{2^{k}}-1)\Rightarrow m^{2}=(2^{2^{k}}-1)i-9\Rightarrow 2^{2^{k+1}}m^{2}=(2^{2^{k}+2^{k+1}}-2^{2^{k+1}})i-9.2^{2^{k+1}}\Rightarrow (m.2^{2^{k}})^{2}\equiv -9(mod 2^{2^{k+1}}-1)$

áp dụng đồng dư cho phần dư sau.

sai rồi bạn ơi! Đoạn cuối bạn suy ra sai rồi. Bạn đọc kĩ lại xem




#532353 chứng minh hoặc bác bỏ khẳng định sau

Đã gửi bởi taideptrai on 08-11-2014 - 17:11 trong Số học

chứng minh hoặc bác bỏ khẳng định sau:

 

luôn tồn tại m là số nguyên dương sao cho

$2^{2^{n}}-1$ là ước của $m^{2}+9$ với mọi n nguyên dương




#526036 P=|(a^{2}-b^{2})(b^{2}-c^{2})(c^...

Đã gửi bởi taideptrai on 24-09-2014 - 23:12 trong Bất đẳng thức - Cực trị

Cho a,b,c không âm có tổng bằng căn bậc hai của 5. Tìm max của P

   

    $P=|(a^{2}-b^{2})(b^{2}-c^{2})(c^{2}-a^{2})|$

 

tớ đã mò ra được max P=$\sqrt{5}$ và quy bài toán về bài toán đơn giản hơn là,

  cho a,b không âm có $a+b\leq\sqrt{5}$

Chứng minh rằng $a^{2}b^{2}(a-b)\leq 1$

 

Ai có cách gì thì đóng góp nhé




#523851 Đề thi Đồng hành cùng Gặp gỡ Toán học.

Đã gửi bởi taideptrai on 10-09-2014 - 22:53 trong Thi HSG cấp Tỉnh, Thành phố. Olympic 30-4. Đề thi và kiểm tra đội tuyển các cấp.

bạn 

hoangtubatu955 có lẽ bạn đã nhầm, bất đẳng thức này không thuần nhất nên không thể chuẩn hóa như bạn được



#520518 có bao nhiêu cách xếp n cái kẹo giống nhau vào k cái hộp giống nhau

Đã gửi bởi taideptrai on 20-08-2014 - 21:40 trong Tổ hợp - Xác suất và thống kê - Số phức

Bài 1: Có bao nhiêu cách để xếp n cái kẹo giống nhau vào k cái hộp giống nhau ( có thể có hộp không có kẹo). n,k nguyên dương

 

Bai 2: Có bao nhiêu cặp tập con không giao nhau của tập hợp n phần tử, n nguyên dương




#519770 làm sao để gả n cô gái cho m chàng trai

Đã gửi bởi taideptrai on 15-08-2014 - 22:31 trong Tổ hợp - Xác suất và thống kê - Số phức

cho n cô gái và m chàng trai.($n\geqm$). Hỏi có bao nhiêu cách gả các cô gái cho các chàng trai sao cho chàng nào cũng có vợ, và mỗi cô chỉ được lấy 1 chồng




#518132 $u_{1}=\frac{1}{2}$ và $u_...

Đã gửi bởi taideptrai on 06-08-2014 - 23:53 trong Dãy số - Giới hạn

ban tinh toan sai roi. dap so phai la 1 moi dung




#469506 phương trình và bất phương trình căn thức

Đã gửi bởi taideptrai on 07-12-2013 - 19:52 trong Phương trình, hệ phương trình và bất phương trình

 
  1. 4X2 _13X+5+​​căn(3X+1)=0
  2. căn(X+3) +2Xcăn(X+1) =2X+căn (X2  +4X+3)
  3. căn (X+2) -2căn(3-X) +3căn(2X+5)-4căn(22-3X)+7=0
  4. 2(X+3X +2)=3căn(X38 +8)
  5. căn(-X2 +7X-6) bé hơn 3+2X

 




#439180 Tìm nghiệm nguyên của phương trình $9x++2=y^{2}+y$

Đã gửi bởi taideptrai on 29-07-2013 - 21:29 trong Số học

Tìm nghiệm nguyên của phương trình $9x++2=y^{2}+y$

 

P/s: các bạn sử dụng phương pháp 'xét số dư' từng vế nha

pt <=> $(y-1)(y+2)=9x$

mà y+2 và y-1 đồng dư khi chia cho 3 và tích của chủng chia hết cho 3 cả 2 số đều chia hết cho 3

=> y= 3k +1  với k nguyên => x=k(k+1) :closedeyes:  :closedeyes:  :closedeyes:




#438300 Viết phương trình đường phân giác của góc BAC

Đã gửi bởi taideptrai on 26-07-2013 - 10:48 trong Hình học

??? Là sao bạn

vẽ hình trên diễn đàn ý.  Nghĩa là phần mềm vẽ hình trên máy tính ấy




#438287 Viết phương trình đường phân giác của góc BAC

Đã gửi bởi taideptrai on 26-07-2013 - 10:00 trong Hình học

Hihi, bài 1 đúng là mình nhầm box thật, cô cùng xin lỗi. Hiện giờ mình chưa học hệ lượng thức nên hk biết làm bài 2 hichic, bài 3 mình cũng làm từ từ mà cũng chưa ra dc.....

dạy tôi vẽ hình với bạn ơi :wub:  :wub:  :wub:




#433714 $\frac{a^{2}}{a+2b^{2}}+...

Đã gửi bởi taideptrai on 08-07-2013 - 10:23 trong Bất đẳng thức và cực trị

vậy thì nếu gặp dạng này thì muốn dùng cauchy-swatch thì nâng bậc à

còn tùy vào giả thiết nữa. Cậu không nên áp đặt như thế !!!! :namtay  :namtay  :namtay  :namtay  :namtay  :namtay




#433706 $\frac{a^{2}}{a+2b^{2}}+...

Đã gửi bởi taideptrai on 08-07-2013 - 10:08 trong Bất đẳng thức và cực trị

uhm . vậy với các dạng này ta nên dùng các bđt thức nào

nếu cậu thích hệ quả của cauchy - swatch thì làm như Ha Manh Huu ấy :icon10:  :icon10:  :icon10:  :icon10:  :icon10:




#433701 $\frac{a^{2}}{a+2b^{2}}+...

Đã gửi bởi taideptrai on 08-07-2013 - 10:03 trong Bất đẳng thức và cực trị

như vậy thì phải làm sa nếu phía trên dùng xvác hả bạn

bất đẳng thúc cậu dùng lúc đầu yếu quá nên không có hiệu quả!!!!! :luoi:  :luoi:  :luoi:




#433697 $\frac{a^{2}}{a+2b^{2}}+...

Đã gửi bởi taideptrai on 08-07-2013 - 09:57 trong Bất đẳng thức và cực trị

$\frac{a^{2}}{a+2b^{2}}+\frac{b^{2}}{b+2c^{2}}+\frac{c^{2}}{c+2a^{2}}\geqslant \frac{(a+b+c)^{2}}{a+b+c+2(a^{2}+b^{2}+c^{2})}= \frac{9}{3+2(a^{2}+b^{2}+c^{2})}$

vậy ta cần chứng minh $a^{2}+b^{2}+c^{2}\leqslant 3$ (1)

giả sử bất đẳng thức vừa nêu là đúng ta có

$(1)\Rightarrow (a+b+c)^{2}\leqslant 3+2(ab+bc+ac)$$\Rightarrow ab+bc+ac\leqslant 3$ 

ta có $ab+bc+ac\leqslant \frac{(a+b+c)^{2}}{3}= 3$ (luôn đúng ) nên (1) đúng 

vậy được đpcm

sai rồi @!!! vì $a^{2}+b^{2}+c^{2}\geq \frac{(a+b+c)^{2}}{3}=3$

chứ không phải$a^{2}+b^{2}+c^{2}\leq 3$   đâu nghen   :angry:  :angry:  :angry:




#433637 $\frac{1}{ac}+\frac{1}{bc}+\frac{1}{ab}$

Đã gửi bởi taideptrai on 07-07-2013 - 21:54 trong Bất đẳng thức và cực trị

mình chém xem sao

Ta có: $\frac{1}{ab}+\frac{1}{bc}+\frac{1}{ca}\geq \frac{9}{ab+bc+ca}=9$

ta sẽ chứng minh vế phải (1)  $\leq 9$

thật vậy $VP^{2}\leq 3(\frac{1}{a^{2}+1}+\frac{1}{b^{2}+1}+\frac{1}{c^{2}+1})\leq 9\Rightarrow VP\leq 3< 9$

$\Rightarrow VT< VP$(BĐT không xảy ra dấu bằng kêr cả có số 3 ở vế phải)

:luoi:  :luoi:  :luoi:  :luoi:  >:)  >:)  >:)  >:)

 sao $3(\frac{1}{a^{2}+1}+\frac{1}{b^{2}+1}+\frac{1}{c^{2}+1})\leq 9$ được nhỉ???????? :wub:  :wub:  :wub:  :wub:  :wub:




#433634 cho x,y,z > 0 và ........

Đã gửi bởi taideptrai on 07-07-2013 - 21:41 trong Bất đẳng thức và cực trị

cho x,y,z > 0 và $x^{2}+y^{2}+z^{2}+2xyz=1.$

chứng minh rằng $xy+yz+zx\leq \frac{1}{2}+2xyz$




#433627 Tìm $Min$ của $A=2x+3y-4z$ và $B=5x-6y+7z$

Đã gửi bởi taideptrai on 07-07-2013 - 21:12 trong Bất đẳng thức và cực trị

Phải có điều kiện của $x,y,z$ khi đó tính $x,y$ theo $z$ rồi thay vào thôi

:namtay  :namtay  :namtay  :namtay  :namtay  :namtay  đúng ý tôi




#433625 $ \sum \frac{a}{1+b^{2}c}\g...

Đã gửi bởi taideptrai on 07-07-2013 - 21:08 trong Bất đẳng thức và cực trị

chứng minh với mọi số dương a,b,c,d thoã mãn a+b+c+d=4

ta có $\frac{a}{1+b^{2}c}+\frac{b}{1+c^{2}d}+\frac{c}{1+d^{2}a}+\frac{d}{1+a^{2}b}\geq 2$

 

a ơi đề là $a+b+c+d=4$ mà

tớ có cách khác xin đóng góp

ta có $\frac{a}{1+b^{2}c}=\frac{a(1+b^{2}c)-ab^{2}c}{1+b^{2}c}=a-\frac{ab^{2}c}{1+b^{2}c}\geq a-\frac{ab^{2}c}{2b\sqrt{c}}=a-\frac{ab\sqrt{c}}{2}$ . tương tự với những cái còn lại 

nên $\sum \frac{a}{1+b^{2}c}\geq a+b+c+d-(\frac{ab\sqrt{c}}{2}+\frac{bc\sqrt{d}}{2}+\frac{cd\sqrt{a}}{2}+\frac{da\sqrt{b}}{2})$

bây giờ ta cần chứng minh $\sum \frac{ab\sqrt{c}}{2}\leq 2$ => $\sum ab\sqrt{c}\leq 4$

 

thật vậy vì$\sum ab\sqrt{c}=\sum \sqrt{a^{2}b^{2}c}\leq \frac{1}{2}(abc+bcd+cda+dab+ab+bc+ca+ad)$

mà $abc+bcd+cda+dab=ac(b+d)+bd(a+c)\leq \frac{(a+c)^{2}}{4}(b+d)+\frac{(b+d)^{2}}{4}(c+a)=(a+c)(b+d)\frac{a+b+c+d}{4}=(a+c)(b+d)\leq \frac{(a+b+c+d)^{2}}{4}=4$

và $ab+bc+cd+da\leq 4$ (dễ dàng chứng minh)

nên $\sum ab\sqrt{c}\leq \frac{1}{2}(abc+bcd+cda+dab+ab+bc+cd+da)\leq 4$

=> đpcm

dấu bằng xảy ra khi a=b=c=d=1




#433395 $\frac{1}{ac}+\frac{1}{bc}+\frac{1}{ab}$

Đã gửi bởi taideptrai on 06-07-2013 - 22:38 trong Bất đẳng thức và cực trị

cho các số dương a,b,c thỏa mãn ab+bc+ac=1 chứng minh rằng

$\frac{1}{ac}+\frac{1}{bc}+\frac{1}{ab}\geq \sqrt{\frac{1}{a^{2}+1}}+\sqrt{\frac{1}{b^{2}+1}}+\sqrt{\frac{1}{c^{2}+1}}$

đẳnh thức xảy ra khi nào

 

cho các số dương a,b,c thỏa mãn ab+bc+ac=1 chứng minh rằng

$\frac{1}{ac}+\frac{1}{bc}+\frac{1}{ab}\geq \sqrt{\frac{1}{a^{2}+1}}+\sqrt{\frac{1}{b^{2}+1}}+\sqrt{\frac{1}{c^{2}+1}}$

đẳnh thức xảy ra khi nà

đề sai hay sao ấy bạn ạ :namtay  :namtay  :namtay  :namtay  :namtay  :namtay

dấu bằng không hề xảy ra khi a=b=c




#433381 Chứng minh $(abc)^2+2a^2+2b^2+2c^2 \ge 7$

Đã gửi bởi taideptrai on 06-07-2013 - 22:04 trong Bất đẳng thức và cực trị

hehehehehe!!! câu b cũng tương tự

 

ta có $a^{2}+b^{2}+c^{2}+(abc)^{2}+1+1\geq a^{2}+b^{2}+c^{2}+2abc+1\geq 2(ab+bc+ca)$

nên $a^{2}+b^{2}+c^{2}+(abc)^{2}+2\geq 2(ab+bc+ca)$

suyra $(a+b+c)^{2}+(abc)^{2}+2\geq 4(ab+bc+ca)$

mà a+b+c=3 => $(abc)^{2}+11\geq 4(ab+bc+ca)$

dấu bằng xảy ra khi a=b=c=1




#433180 Tìm GTLN và GTNN của biểu thức A = x + y + z. Biết $y^{2}+ yz...

Đã gửi bởi taideptrai on 06-07-2013 - 09:23 trong Bất đẳng thức và cực trị

Theo đề bài, biến đối ta được 

$A^{2}= 2-(x-y)^{2}-(x-z)^{2}$.

Vì $(x-y)^{2}+(x-z)^{2}\geqslant 0$ nên $A^{2}\leqslant 2$.

Dâú bang xay ra khi và chi khi x = y = z = $\pm \frac{\sqrt{2}}{3}$

=> MaxA =$ \sqrt{2}$ <=> x = y = z = $\frac{\sqrt{2}}{3}$

 

MinA =-$ \sqrt{2}$ <=> x = y = z = -$\frac{\sqrt{2}}{3}$

??????? >:)  >:)  >:)

phần mềm gõ latex bị sao vậy???




#433179 Chứng minh BĐT: $a+b^{2}+c^{3}-ab-bc-ca\leqslan...

Đã gửi bởi taideptrai on 06-07-2013 - 09:21 trong Bất đẳng thức và cực trị

Từ gt suy ra $(a-1)(b-1)(c-1)\leq 0$

$\Rightarrow abc+a+b+c-(ab+bc+ac)\leq 1$

Lại có $0\leq b\leq 1$ nên $b^2\leq b$ $(1)$ 

$0\leq c\leq 1\Rightarrow \left\{\begin{matrix}c^3\leq c^2 & & \\ c^2\leq c & & \end{matrix}\right.$

$\Rightarrow c^3\leq c$ $(2)$ 

Mà $abc\geq 0$ $(3)$

Từ $(1) ; (2) ; (3)$ suy ra $a+b^2+c^3-(ab+bc+ac)\leq a+b+c+abc-(ab+bc+ac)\leq 1$

rất hay nhưng tớ bổ sung thêm đk dấu bằng xảy ra khi trong 3 số có 2 số bằng 1 và 1 số bằng 0




#433140 Tìm $Min$ của $A=2x+3y-4z$ và $B=5x-6y+7z$

Đã gửi bởi taideptrai on 05-07-2013 - 23:11 trong Bất đẳng thức và cực trị

thiếu đk của x,y,z kìa em ơi!!! :luoi:  :luoi: